LSAT and Law School Admissions Forum

Get expert LSAT preparation and law school admissions advice from PowerScore Test Preparation.

 Jay Donnell
PowerScore Staff
  • PowerScore Staff
  • Posts: 144
  • Joined: Jan 09, 2019
|
#63297
Hi oli!

Yes, you're right in that A provided much too narrow of a principle to be used in this situation.


To use a principle, we must ensure that the details of our situation allow us to "activate" the given principle we seek to use for support.


For example, let's say that I am contemplating going to the beach on a day that could be rainy, but I decided that I should not go because there is a chance of rain.

Would I be able to help that argument with a rule that says: You should never go to the beach on any day that is guaranteed to rain?

Of course not, because all we know is that it could rain.

In this case, we know that "Once people see the destructive consequences of extreme freedom, they may prefer to establish totalitarian political regimes." That fact certainly doesn't guarantee that extreme freedom will inevitably lead to the establishment of a totalitarian political regime, so the principle in A is too narrow/strong to be useful here.

The correct response in D provides language that is much more useful, as we certainly know enough to "activate" a principle that just needs to know that this system could lead to totalitarianism.

I hope that helps!
User avatar
 Blondeucus
  • Posts: 9
  • Joined: Jan 13, 2023
|
#102143
Can someone explain why C is wrong?
 Luke Haqq
PowerScore Staff
  • PowerScore Staff
  • Posts: 742
  • Joined: Apr 26, 2012
|
#102147
Hi Blondeucus!

To understand why (C) is incorrect and (D) correct, let's start with the conclusion: "one should not support political systems that allow extreme freedom."

Why does the author conclude this? The author reasons that extreme freedom can lead to destructive consequences, and seeing those destructive consequences may lead people to prefer to establish totalitarian regimes. There's a gap missing between those premises, though, and the conclusion's normative recommendation about what one should not support. The author seems to be assuming that one should not support something that might lead people to prefer totalitarian regimes.

This is why (D) is correct: "One should not support any political system whose destructive consequences could lead people to prefer totalitarian political regimes." This connects the chain of reasoning in the first few sentences with the normative "should" statement in the conclusion.

Answer choice (C) states, "One should support only those political systems that give people the freedom to make wise choices." We don't know about how wise choices factor into the stimulus. It's possible that a political system that allows for extreme freedom also allows for the freedom to make wise choices. If that's the case, then plugging (C) into the stimulus wouldn't allow the conclusion to follow from the premises.

Get the most out of your LSAT Prep Plus subscription.

Analyze and track your performance with our Testing and Analytics Package.